1
$\begingroup$

Is there any exact formula or at least exact inequalities for the following intehral

$$ \int_{2}^{x}{{\rm d}t \over \left\lfloor\vphantom{\large h}% \log(x)/\log(t)\right\rfloor \log\left(t\right)} $$

where $\lfloor x\rfloor$ is the greatest integer less than or equal to $x$.

added:

When I use

$$ x-1<\lfloor x\rfloor\le x $$ I get $$ \frac{x-2}{\log (x)}=\int_2^x\frac{{\mathrm d}t}{\log (x)}\leq \int_2^x\frac{{\mathrm d}t}{\left\lfloor\vphantom{\large h}\log (x)/\log (t)\right\rfloor\log (t)}\le \int_2^x\frac{\mathrm dt}{\log (x)-\log (t)} $$ but they are not exact enough. I need more closer bounds.

$\endgroup$
5
  • $\begingroup$ Why not replace log t by a (temporary) constant c, and then sum rectangle areas? Gerhard "Ask Me About System Design" Paseman, 2012.10.25 $\endgroup$ Oct 25, 2012 at 15:59
  • $\begingroup$ To answer my own question, because it is dt, not dx. Gerhard "So Sorry About That, Chief" Paseman, 2012.10.25 $\endgroup$ Oct 25, 2012 at 16:03
  • $\begingroup$ Why do you integrate from $2$ and not for $e$? $\endgroup$ Oct 25, 2012 at 16:14
  • $\begingroup$ @Davide Giraudo, @Gerhard Paseman, I added some phrases. $\endgroup$
    – asd
    Oct 25, 2012 at 19:19
  • $\begingroup$ Can you use z-(1/2+atan(tan(pi*(z-1/2)))/pi) instead of the floor function, replacing z with log(x)/log(t)? Such a thing may not necessarily work and could introduce other problems, but perhaps it could help? $\endgroup$
    – user78169
    Aug 13, 2015 at 22:17

1 Answer 1

5
$\begingroup$

Note that $\left\lfloor \dfrac{\log x}{\log t} \right\rfloor = n$ for $x^{1/(n+1)} < t < x^{1/n}$, so if $m = \left\lceil \dfrac{\log x}{\log 2} \right \rceil$

$$ \eqalign{\int_{2}^x &\frac{dt}{\left\lfloor \frac{\log x}{\log t}\right\rfloor \log t} = \int_2^{x^{1/m}} \dfrac{dt}{m \log t} + \sum_{j=1}^{m-1} \int_{x^{1/(j+1)}}^{x^{1/j}} \dfrac{dt}{j \log t} \cr &= \frac{1}{m} \text{Ei}(1,-\log 2) - \text{Ei}(1,-\log x)+ \sum_{j=2}^{m} \frac{1}{j(j-1)} \text{Ei}(1,-\frac{1}{j} \log x) \cr}$$

You might also note that $$ \frac{x^{1/j} - x^{1/(j+1)}}{\log x} \le \int_{x^{1/(j+1)}}^{x^{1/j}} \dfrac{dt}{j \log t} \le \frac{j+1}{j} \frac{x^{1/j} - x^{1/(j+1)}}{\log x} $$

so using these bounds for all $j > N$ and the exact values for $j \le N$ will give you approximations with relative error at most $1/N$.

$\endgroup$

Your Answer

By clicking “Post Your Answer”, you agree to our terms of service and acknowledge you have read our privacy policy.

Not the answer you're looking for? Browse other questions tagged or ask your own question.